Feynman gobierna con estados de helicidad.

  • Cada vez que se establecen las reglas de Feynman, nunca se mencionan las helicidades; esto me parece muy confuso. ¿Cómo se introduce y explica eso?

  • ¿Existe un argumento intuitivo / simple de por qué las partículas sin masa deberían tener "helicidades" (y no polarizaciones) y solo pueden ser de la forma ±  algún entero positivo ? (... he visto algunos argumentos muy detallados que dependen de la teoría de la representación para el pequeño grupo de partículas sin masa y varias otras consideraciones topológicas; aquí estoy buscando una explicación "rápida" para eso...)

  • ¿Hay alguna razón por la que las amplitudes de dispersión de gluones polarizados a nivel de árbol puedan escribirse "obviamente" de alguna manera? Como por ejemplo, considere un proceso donde dos gluones de helicidad positiva de momentos pag 1 y pag 2 dispersarse en dos gluones de helicidad negativa de momentos pag 3 y pag 4 entonces, a nivel de árbol, la amplitud de dispersión es,

A ( pag 1 + , pag 2 + , pag 3 , pag 4 ) = i gramo 2 4 pag 1 . pag 2 ϵ 2 + ϵ 3 ( 2 pag 3 . ϵ 4 ) ( 2 pag 2 . ϵ 1 + )

dónde ϵ i ± es la polarización de la i t h partícula.

En algunos lugares he visto esta expresión escrita casi directamente. ¿Es lo anterior de alguna manera obvio?

helicidad ( h = S pag ^ ) solo es útil para partículas sin masa porque para ellas es Invariante de Lorentz. Para partículas con masa, puede encontrar una transformación de Lorentz (simplemente viaje en la misma dirección que la partícula pero más rápido) tal que pag ^ pag ^ . Entonces el h para partículas masivas depende del marco. Las helicidades son ± algún (medio) entero por la misma razón que el giro toma valores (medio) entero. Para eso necesitas entender las representaciones de S tu ( 2 ) . Eso debería responder al segundo punto al menos.
@Kyle Supongo que mi pregunta sobre la restricción numérica de los valores de helicidad no estaba clara. Parece funcionar a diferencia de la teoría de la representación de S tu ( 2 ) . Una partícula en el espín s representacion de S tu ( 2 ) puede tener todos los estados de espín desde s a s en pasos de 1 dónde s está restringido a ser un (medio) entero. PERO NO es así como funciona la helicidad: las helicidades solo pueden ser ± ( h a yo F ) i norte t mi gramo mi r . Quería saber si hay una explicación "rápida" para eso.
Creo que así es como funciona la helicidad ... si estás pensando en un giro sin masa 1 / 2 partículas, entonces claramente la helicidad será sólo por ± 1 / 2 . Para partículas de espín 1 sin masa, en principio podría pensar que la helicidad podría ser 0 , ± 1 pero el 0 el modo resulta invariancia del indicador de ruptura, por lo que la helicidad es la ± 1 . Si tuviera que observar los estados de helicidad para el espín sin masa 3 / 2 partículas, tendrías ± 3 / 2 , ± 1 / 2 .
@Kyle ¿Puede explicar amablemente por qué la helicidad funciona de esta manera? Quiero saber de una "prueba rápida" de esto: que para un giro sin masa 1 2 la helciidad tiene que ser ± 1 2 y para giro sin masa 3 2 la helicidad tiene que ser $\pm \frac{3]{2}$ etc..

Respuestas (2)

El argumento para la primera pregunta es el siguiente:

Considere el vector de Pauli-Lubanski W m = ϵ m v ρ σ PAG v METRO ρ σ . Dónde PAG m son los momentos y METRO m v son los generadores de Lorentz. (La norma de este vector es un casimiro del grupo de Poincaré, pero este hecho no será necesario para el argumento).

Por consideraciones de simetría tenemos W m PAG m = 0 . Ahora, en el caso de una partícula sin masa, un vector ortogonal a un vector similar a la luz debe ser proporcional a ella (ejercicio fácil). De este modo W m = h PAG m , ( h = C o norte s t . ). Ahora, la componente cero del vector de Pauli-Lubanski viene dada por:

W 0 = ϵ 0 v ρ σ PAG m METRO m v = ϵ a b C PAG a METRO b C = PAG . j , (donde la suma después de la segunda igualdad es solo en los índices espaciales, y j son los generadores de rotación).

Por lo tanto, la constante de proporcionalidad h = W 0 PAG 0 = PAG . j | PAG | es la helicidad.

Ahora, en el nivel cuántico, si rotamos un ángulo de 2 π alrededor del eje de cantidad de movimiento, la función de onda adquiere una fase de: mi X pag ( 2 π i PAG | PAG | . j ) = mi X pag ( 2 π i h ) . Este factor debe ser ± 1 de acuerdo con las estadísticas de partículas por lo tanto h debe ser medio entero.

En cuanto a la segunda pregunta, un método muy poderoso para construir las amplitudes de los gluones es el enfoque del twistor. Consulte el siguiente artículo de NP Nair para obtener una exposición clara.

Actualizar:

Esta actualización se refiere a las preguntas formuladas por el usuario 6818 en los comentarios:

Para simplificar, consideraré el caso de un fotón y no de gluones.

La estrategia de la solución se basa en la construcción explícita del momento angular y el espín de un campo de fotones libres (que dependen de los vectores de polarización) y muestra que las relaciones anteriores se cumplen para el campo de fotones. El momento del fotón y las densidades del momento angular se pueden obtener mediante el teorema de Noether a partir del Lagrangiano del fotón. Alternativamente, es bien sabido que el momento lineal del fotón viene dado por el vector de Poynting (proporcional a) mi × B , y no es difícil convencerse de que la densidad de momento angular total es (proporcional a) X × mi × B .

Ahora, el momento angular total se puede descomponer en momentos angulares y angulares de espín (consulte KT Hecht: mecánica cuántica (página 584 ecuación 16))

j = d 3 X ( X × mi × B ) = d 3 X ( mi × A + i = 1 3 mi j X × A j )

El primer término del lado derecho se puede interpretar como el giro y el segundo como el momento angular orbital, ya que es proporcional a la posición.

Ahora, ni el espín ni las densidades del momento angular orbital son invariantes de calibre (solo su suma lo es). Pero, se puede argumentar que el momento angular orbital total es cero porque la posición promedia cero, por lo tanto, el giro total:

S = d 3 X ( mi × A )

es calibre invariante:

Ahora, podemos observar que en la cuantización canónica: [ A j , mi k ] = i d j k , obtenemos [ S j , S k ] = 2 i ϵ j k yo S yo . Cuáles son las relaciones de conmutación del momento angular aparte del factor 2.

Ahora, reemplazando la solución de onda plana:

A k = k , metro = 1 , 2 a k metro ϵ metro ( k ) mi X pag ( i ( k . X | k | t ) ) + h . C .

(La condición ϵ metro ( k ) . k = 0 , es sólo una consecuencia de la desaparición de las fuentes).

Obtenemos:

S = k , metro = 1 , 2 ( 1 ) metro a k metro a k metro k ^ = k ( norte 1 norte 2 ) k ^

(dónde norte 1 , norte 2 son los números de fotones polarizados circularmente a la derecha y a la izquierda). Por lo tanto, para un solo fotón libre, el giro total, por lo tanto, el momento angular total se alinean a lo largo o en oposición al momento, que es el mismo resultado establecido en la primera parte de la respuesta.

En segundo lugar, los operadores de espín total de fotones existen y se transforman (hasta un factor de dos) como operadores de momento angular de espín 1/2.

Gracias por tu esclarecedora respuesta. ¿Hay un error tipográfico en su definición de W m ? como no debería ser W m = ϵ m v ρ σ METRO v ρ PAG σ ? ¿Puede explicar amablemente cómo su definición de "helicidad" corresponde a la "otra" forma de pensar como la 4 -vectores ϵ m ( pag ) que se requiere para describir un fotón de impulso pag como A m ( pag ) = ϵ m ( pag ) mi i pag v X v ? (... y uno puede elegir un calibre tal que ϵ 0 = 0 y ϵ m pag m = 0 ..) ¿Y si pudiera explicar por qué no existe una noción de "spin" para partículas sin masa?
Conozco la explicación de por qué no hay giro para partículas sin masa al pensar en cuál es su pequeño grupo: es S O ( 3 ) partículas masivas y S O ( 2 ) sin masa y, por lo tanto, espín para el primero y helicidad para el segundo. Pero estoy preguntando si hay una explicación intuitiva "rápida" y si puede reconciliar estas imágenes diferentes, como ver mis comentarios sobre la respuesta de Sidious Lord.
He agregado una actualización a la respuesta, en la que he corregido el error de índice y he agregado una explicación sobre la relación de la helicidad con los vectores de polarización (no basada en la clasificación de Wigner). Además, he agregado una explicación sobre el giro del fotón.

Cada vez que se establecen las reglas de Feynman, nunca se mencionan las helicidades; esto me parece muy confuso. ¿Cómo se introduce y explica eso?

En QFT puede representar el estado de un quanta de calibre por su impulso y helicidad. También puede hacerlo de forma dependiente del calibre especificando el impulso y un vector de polarización ϵ m ( pag ) . esto es nulo ϵ ( pag ) 2 = 0 y está sujeto a una equivalencia de calibre ϵ m ( pag ) ϵ m ( pag ) + pag m . Cuando calcula una amplitud de dispersión usando las reglas de Feynman, la forma en que describe los estados de las partículas externas es usando vectores de polarización. Este es material de libro de texto estándar, así que no veo qué te confunde.

¿Existe un argumento intuitivo / simple de por qué las partículas sin masa deberían tener "helicidades" (y no polarizaciones) y solo pueden tener la forma ± algún número entero positivo? (... he visto algunos argumentos muy detallados que dependen de la teoría de la representación para el pequeño grupo de partículas sin masa y varias otras consideraciones topológicas; aquí estoy buscando una explicación "rápida" para eso...)

Matemáticamente, las partículas están en correspondencia con representaciones irreducibles del grupo de Lorentz. La teoría de la representación del grupo de Lorentz es un poco delicada porque no es compacta. Pero la falta de compacidad es fácil de entender físicamente: se debe a que uno puede aumentar en una cantidad arbitraria. Así que olvidémonos de los impulsos y miremos solo las transformaciones de Lorentz que preservan la dirección del impulso. Debería poder demostrar que estas transformaciones forman un S O ( 2 ) tu ( 1 ) subgrupo. Actúan sobre los estados multiplicándolos por una fase. El cargo bajo este tu ( 1 ) grupo es sólo la helicidad.

¿Hay alguna razón por la que las amplitudes de dispersión de gluones polarizados a nivel de árbol puedan escribirse "obviamente" de alguna manera?

¿Qué quieres decir con obviamente? Es fácil escribirlo a nivel de árbol, es un ejercicio típico de QFT en el uso de las reglas de Feynman. Y por cierto, me parece que la fórmula que escribiste no puede ser correcta ya que rompe la invariancia del indicador. Debe ser invariante bajo ϵ m ( pag ) ϵ m ( pag ) + pag m .

Gracias por la respuesta. Soy consciente de la definición de helcity que das usando la teoría de la representación. Pero esta definición es difícil de conciliar con la dada a través de ϵ m o el que está en términos del vector de Pauli-Lubansky (como la respuesta anterior). Sería genial si pudieras ayudar a conectar estas 3 imágenes "diferentes" de helicity. Y también si puede dar una explicación "rápida" de su imagen de por qué la helicidad debería ser ± ( h a yo F ) i norte t mi gramo mi r .
Acerca de las reglas de Feynman con helicidad: supongo que mi pregunta no fue clara. Digamos que tengo un vértice de 3 gluones donde los tres gluones con todos los momentos entrantes están etiquetados por el triplete ( C o yo o tu r   i norte d mi X , L o r mi norte t z   i norte d mi X , metro o metro mi norte t tu metro ) como ( a , m , pag 1 ) , ( b , v , pag 2 ) , ( C , λ , pag 3 ) entonces el factor de vértice para esto es i gramo F a b C [ η m v ( pag 1 pag 2 ) λ + η v λ ( pag 2 pag 3 ) m + η λ m ( pag 3 pag 1 ) v ] - esto es estándar. Ahora, ¿puede decirme cómo en lo anterior incorporo las helicidades de los tres gluones si eso también se especifica?
@ user6818 Su pregunta sobre la relación entre el vector de polarización ϵ m , el vector de Pauli-Lubansky y la helicidad es mucho más precisa que su pregunta inicial, por lo que también puedo ser más preciso en mi respuesta. La forma más rápida que conozco de conectar los vectores de helicidad y polarización utiliza el lenguaje de helicidad del espinor. Para cualquier vector en 4D podemos formar la siguiente cantidad v α α ˙ = v m σ α α ˙ m , dónde σ m = ( 1 , σ ) y σ son las matrices de Pauli. [ver siguiente...]
[...continuación] Entonces, un momento similar a la luz se puede escribir en forma factorizada pag α α ˙ = λ α λ ~ α ˙ (¡demuestra esto!). El grupito actúa por multiplicación por una fase: λ mi i ϕ λ y λ ~ mi i ϕ λ ~ . Entonces supongamos que quiero escribir alguna función de onda para un giro sin masa 1 2 partícula. En el espacio de impulso, esto será solo λ por helicidad 1 2 y λ ~ por helicidad 1 2 (o lo contrario, dependiendo de las convenciones) [ver siguiente...]
[...continuación] La regla es que para una partícula de helicidad h la función de onda se transforma por una fase mi 2 i h ϕ bajo una rotación por ϕ que conserva el impulso. Ahora, para una partícula de espín 1 las cosas se ponen más interesantes. Si lo piensas un poco, puedes escribir una función de onda que se transforme como mi 2 i ϕ pero va a ser de la forma λ α λ β . No es de tipo vectorial (los vectores tienen un índice sin puntos y otro con puntos), pero en factor corresponde a la parte anti-autodual de un tensor de rango dos. [ver siguiente...]
[...continuación] Este tensor de rango dos es, de hecho, la intensidad de campo del campo de norma. Pero podemos escribir un vector que se transforme de la manera correcta, sin embargo, dado que este es un campo de calibre, contendrá alguna dependencia de calibre. Entonces, introduzcamos algunos espinores m y m ~ y usarlos para escribir ϵ α α ˙ + = λ α m ~ α ˙ [ λ ~ m ~ ] y una expresión similar para ϵ α α ˙ . ¿Qué pasa cuando cambias los espinores? m y m ~ ? ¡Obtienes transformaciones de calibre!
Gracias por la explicación.. Más o menos he visto este formalismo. ¿Puede explicar amablemente cuál es el significado físico de este λ α y λ ~ α ˙ ? ¿Son los espinores de Weyl de dos componentes que representan su partícula de espín 1/2 sin masa?
Y no entendí cómo surge el vector de Pauli-Lubanski o la restricción en el valor numérico de la helicidad en este formalismo. ¿Y puede usted amablemente motivar su definición de ϵ α α ˙ + en tu ultimo comentario?